answer answer answer answer

Answer Answer Answer Answer

Answers

Answer 1
All you do is pythagorean theorem. Which is A squared plus B squared equals C squared. Think of each graph as a right triangle. If you make it a triangle then you know that one side is 6 and one side is 2 so then you can figure out the problem by using Pythagorean theorem to solve for the long side also known as C or C squared
Answer Answer Answer Answer

Related Questions

25 points

Solve for x, y, and z.

Answers

Answer:

Step-by-step explanation:

x=57

y=57

z=66

Rich weighed 320 pounds on June 1st. He lost 3 pounds per week by following a meal plan provided by a doctor for w Weeks write a function for R, Rich's weight. ​

Answers

Function for R=320-3w

The sum of two numbers is __________ greater than either number.
a) Always
b) Sometimes
c) Never
d) Not enough information

Answers

A) Always because you’re adding them up

The correct answer is b) Sometimes.

Given that a statement,

"The sum of two numbers is __________ greater than either number."

We need to fill the blank from the options provided.

The sum of two numbers can be greater than either number depending on the specific values of the numbers.

Let's consider some examples:

Example 1: If we have two positive numbers, such as 3 and 4, their sum (3 + 4 = 7) is greater than either number individually.

Example 2: If we have one positive number and one negative number, such as 5 and -8, their sum (5 + (-8) = -3) is greater than -8 but less than 5.

Example 3: However, if we have two negative numbers, such as -2 and -4, their sum (-2 + (-4) = -6) is not greater than either number individually.

Therefore, since the statement holds true in some cases but not in others, the correct answer is sometimes (b).

Learn more about Sum click;

https://brainly.com/question/31538098

#SPJ6

Using the triangle in the diagram, what would be the coordinate of A' if reflected over the line x=2 ? *

(0, 3)

(0, -3)

(4, -3)

(-4, 3)

Answers

It would be (0,3). You can see that the A is 2 units from the line. If you do it from the other side, then it would line up to (0,3)

tistics
-2
1 X
--
»
--6-
2 of 8
© The mean temperature for the first 4 days in January was -4°C.
The mean temperature for the first 5 days in January was -6°C.
What was the temperature on the 5th day?​

Answers

Answer: The temperature on the fifth day is -14°C

Step-by-step explanation:

When we have a set of N elements, that can be written as:

{a₁, a₂, ..., aₙ}

The mean of these N elements is be written as:

Mₙ = (a₁ + a₂ + ... + aₙ)/N

Now, when we take in account the temperature for the first four days of January, we will have a set of 4 elements, {a₁, a₂, a₃, a₄}

Where each one of these elements represents the temperature of one particular day.

The mean temperature for the first four days will be:

M₄ = (a₁ + a₂ + a₃ + a₄)/4

And we know that this is equal to 4°C

Now we also could do the next replacement:

(a₁ + a₂ + a₃ + a₄) = T

Then we can write:

M₄ = -4°C =  (a₁ + a₂ + a₃ + a₄)/4 = T/4

Then we can solve:

-4°C = T/4

-4°C*4 = T

-16°C = T

This is the sum of the temperatures for the first four days of January.

Now we know that the mean for the first five days is -6°C

Then we have:

M₅ = (a₁ + a₂ + a₃ + a₄ + a₅)/5 = -6°C

Where a₅ is the temperature on the fifth day of January, which is the value we want to find.

And we know that:

a₁ + a₂ + a₃ + a₄ = -16°C

We can replace that in the equation to get:

(-16°C + a₅)/5 = -6°C

(-16°C + a₅) = 5*(-6°C) = -30°C

a₅ = -30°C + 16°C = -14°C

a₅ = -14°C

The temperature on the fifth day is -14°C

Help me pls pls pls pls

Answers

Answer:

sorry but all i got from this is the first one should be -3

Step-by-step explanation:

sorry that i couldn't answer all :)

divide: 3.78 by 4.6 with explaination​

Answers

Answer:

189/230

Step-by-step explanation:

Convert these to fractions first, because then it would be easier to calculate.

3 39/50 divided by 4 3/5

Turn the fractions into improper fractions:

189/50 divided by 23/5 = 189/50 times the reciprocal of 23/5

So we have:

189/50 x 5/23 = 945/1150 = 189/230

Which answer represents the expression in its expanded form?

Answers

the answer is 6x^2-2x-20

Which equation, solved for x, is equivalent to y-h=x-g/k

A.x=k(y-h)-g

B.x=ky -h+g

C.x=k(y-h) +9

D.x = ky-h-9

Answers

Answer:

a

Step-by-step explanation:

The equation equivalent to the given equation y-h=x-g/k is .x=k(y-h) +g. The correct option is C.

What is an expression?

The mathematical expression combines numerical variables and operations denoted by addition, subtraction, multiplication, and division signs.

Mathematical symbols can be used to represent numbers (constants), variables, operations, functions, brackets, punctuation, and grouping. They can also denote the logical syntax's operation order and other properties.

Given that the expression is y-h=x-g/k. The equation will be solved as,

y-h = ( x-g ) / k

k( y-h ) = x - g

k( y-h ) + g = x

Therefore, the equation equivalent to the given equation y-h=x-g/k is .x=k(y-h) +g. The correct option is C.

To know more about an expression follow

https://brainly.com/question/11630502

#SPJ2

Someone plz help me I will give brainliest!!

Answers

It’s pretty easy it’s 35

Answer:

it's 35

Step-by-step explanation:

3+5 = 8

35 is odd

and 3-5 = 2

I don’t know the answer and I’m doing a text I need help!

Answers

Answer:

your answer will be A

Step-by-step explanation:

^_^

The value of a boat is $23,400. It loses 10% of its value every year. Find the approximate monthly percent decrease in value. Round your answer to the nearest hundredth of a percent.

Answers

Answer:

Value of boat = $23,400

Loss of value by boat per year = 8%

To find: -  Monthly percent decrease in value of boat.  

Solution: - Decrease of value per year = 8% of $23,400 = $1,872. Monthly decrease in value = $1,872/12 = $156. Monthly percentage decrease = ($156/$23,400) * 100 = 0.6667 or 0.67 (rounded to nearest hundredth)

Step-by-step explanation:

Hannah and Anthony are siblings who have ages that are consecutive odd integers. The sum of their
ages iS 92. Which equations could be used to find Hannah's age, h, if she is the older sibling?

Answers

Answer:

hannah is 47 and Anthony is 45

Step-by-step explanation:

Sum= 92= a+(a+1)

sum= 45+(46+1)

sum= 45+47

sum= 92

Which of these represents slope-intercept form of a linear equation?Immersive Reader
(1 Point)

y - y1 = m(x-x1)

Ax + By = C

y=kx

y = mx + b

Answers

The correct answer is A
Slope-intercept of an equation of is y=mx+b

The shadow cast by a lampshade has a boundary in the shape of a hyperbola defined by the equation StartFraction (y minus 3) squared Over 2 EndFraction minus StartFraction x squared Over 1 EndFraction = 1. What could the center of the hyperbola represent?


the location of the light bulb

the bottom of the lampshade

the top of the lampshade

a vertex of the shadow

Answers

Answer:

A THE LOCATION OF THE LIGHT BULB

I got this right on the assignment

Step-by-step explanation:

The centre of the hyperbola represent will represent the location of the light bulb.

What is the centre of a hyperbola?

The centre of a hyperbola is the midpoint of the line segment joining its foci. The transverse axis is the line segment that contains the centre of the hyperbola and whose endpoints are the two vertices of the hyperbola.

As we know that the centre of a hyperbola is the midpoint of the line segment joining its foci. Now, if we plot the hyperbola with equation

[tex]\dfrac{(y-3)^2}{2}-\dfrac{x^2}{1}=1[/tex]

The hyperbola will look like the one given in the below image, further if draw two lines to know the centre of the hyperbola, we will find that the centre of the hyperbola will lie at coordinate (0,3).

Further, the centre of the hyperbola represents the light bulb while the hyperbolas will represent the shadow cast by the lampshade.

Hence, the centre of the hyperbola represent will represent the location of the light bulb.

Learn more about Hyperbola:

https://brainly.com/question/12919612

Have you ever devised a way to solve a problem? How?

Answers

Answer:

What kind of problem?

Step-by-step explanation:

Work out the following, giving your answers in their simplest form:
a) 2/3 divided by 1/4
b) 7 divided by 2/3

Answers

definitely recommend the app Fraction Calculator Plus

a) 2 and 2/3
b) 10 and 1/2

a)

who is correct Alex or sophia and why?​

Answers

Answer:

alex

Step-by-step explanation:

Sophia is correct !!!!!!!!

Someone help please

Answers

8-5 = 3
17^3
17+3 = 20
20 is the answer

find x and y!!!!!!!!!!

Answers

Answer:

x = 8

y = 8√3

Step-by-step explanation:

sin(30) = 1/2

cos(30) = √3/2

They are rule!!!

sin = opposite / hypotenuse


sin 30° = x / 16

16 sin 30° = x

x = 16 sin 30°

x = 8



sin 60° = y / 16

16 sin 60° = y

y = 16 sin 60°

y = 13.85640....

y = 13.9 (nearest tenth)

please and thank you ​

Answers

Answer: 6 units

Step-by-step explanation:

Since this is just a reflection, the length of A'C' shouldn't be different from the length of AC.

Andre and Noah started tracking their savings at the same time. Andre started with $15 and deposits $5 per week. Noah started with $2.50 and deposits $7.50 per week. (ill give brainiest and extra 50 points)

Answers

Answer:

Andre     Noah     Weeks

 20           10             1

 25         17.50         2

 30           25            3

Step-by-step explanation:

what is 4.2j+6.7j-5j-5​

Answers

Answer: 5.9j-6

Step-by-step explanation:

can someone help please ?

Answers

Answer:

5 is the hight

Step-by-step explanation:

the height is 5!!!!!

Which number line shows the solution of 5x - 25 < -15?
A.
-100 B 7 B
.
43
-2
.1
12
3 4 5 6 7 8 9 10
B.
-10.9 7 6 5 4 3 2
-1 0
1 2
3
4 5
6 7 8 9 10
O
20 -19 - 18 - 17 -16 -15 -14 -13 -12 -10 -987
43210
-109876
6 7 8 9 10

Answers

I think a. But I don’t understand this question.

Find the slope (-2,1)and (1,2.5)

Answers

Answer:

.5

Step-by-step explanation:

slope = rise/run= 1.5/ 3= 0.5

hope that helps if it does please mark as brainliest.

HELP!! Find the value of x in the isosceles triangle shown below.

Answers

i think the answer is 10

if alpha and beta are the zeros of the quadratic polynomial f(x)=2x^2+11x+25. Find alpha^4+beta^4 and 1/alpha+1/beta-2alpha*beta​

Answers

Step-by-step explanation:

[tex]2 {x}^{2} + 11x + 5 \\ {x}^{2} + 5.5x + 12.5 \\ = -2.75-2.222i \\ \: \: \: - 2.75 + 2.22i \\ \alpha = - 2.75 - 2.22i \\ \beta = - 2.75 + 2.22i[/tex]

[tex] { \alpha }^{4} + { \beta }^{4} = -284.2912344[/tex]

[tex]\frac{1}{ \beta } + \frac{1}{ \alpha } + 2 \alpha \beta = 24.9818[/tex]

evaluate the function when x = -3

Answers

Answer:

10

Step-by-step explanation:

f(-3)=-2(-3)+4=6+4=10

5)
D
-10 + 7x
70°
E
12x + 10
А с
A) 12
C) 8
B) 10
D) 2

Answers

Answer:

B) 10

Step-by-step explanation:

Exterior angle property: Exterior angle equals the sum of opposite interior angles.

12x + 10 = -10 + 7x +  70 {Subtract 10 from both sides}

12x        = -10 + 7x + 70 - 10        

       12x = -10 + 70 - 10 + 7x  {Combine like terms}

      12x  = 50 + 7x          {Subtract 7x from both sides}

12x - 7x = 50

       5x = 50 {Divide both sides by 5}

         x = 50/5

x = 10

Other Questions
help pleaseeeeeee!!!!!! I need help pls. I will give brainiest Aqueous sodium hydroxide forms a light blue precipitate. What is the formula of the blue precipitate? 120g:5kg in simplest form 4. What steps can we take to ensure that we can leave a better world for future generations ?Help !! Samantha is accused of selling drugs. When she appears before the judge, she asks for a lawyer because she cant afford one herself. The judge denies her access to a lawyer.A. 1st AmendmentB. 4th AmendmentC. 6th AmendmentD. 7th Amendment 8(__+__)=8m+8n Please help, 7 cm10.5 cm13 cmYour Turn!3 m15 m11 m5m Can you help me with this linear equation? Find x:-8x+7=4 Which form of birth control could people use without altering hormonal systems or blocking sperm from fertilizing an egg? Past or present racist heath care laws? Upload your Digital Video Revolution project. If , then what is mto the nearest degree? What effect does the transfer of matter and energy have on organism and theirenvironment? at what rate of compound interest per annum will a sum of rupees 2500 become rupees 2500 in 2 years The diagram shows the positionof three towns represented byletters A, B and C.NAB is 7 km from A on a bearing of 037C is 8 km from B on a bearing of 140Find the bearing of C from A.Give your answer correct to 1 decimal place. 5 donuts and 2 cups of hot Chocolate costs 24$. 2 donuts and 3 cups of hot chocolate costs 14$.How much does each donut and each cup of hot chocolate cost Pls help w my spanish!! An inequality is shown. Select all the values of n that make the inequality true.2/3n3/2 plss helppp The circumference of a circle is 9.31cm. Find the length of the diameter. Give your answer rounded to 2 DP.